Chapter 6- Disorders of the Breasts (ob)

Réussis tes devoirs et examens dès maintenant avec Quizwiz!

When counseling a patient on how to prepare for her first mammogram, what should the nurse include?

"Don't wear any powder, deodorant, or jewelery."

When counseling a woman with monthly breast pain, what dietary recommendations should the nurse provide?

"Eat plenty of fruits, vegetables, and whole grains, and follow a low-fat diet."

The nurse is teaching a client with mastitis about care measures. Which client statement indicates effective teaching?

"I can use warm soaks to my breast to relieve the discomfort."

After teaching a woman about breast self-examination, which statement indicates that the teaching was successful?

"I'll do the check a week after my period."

The nurse is giving an educational event for a local women's group on self-examination of the breast. The nurse tells the women's group to raise their arms and inspect their breasts in a mirror. A member of the women's group asks the nurse why she needs to do this. What is the nurse's best response?

"It will help to observe for dimpling."

It is recommended that a 48-year-old female client with breast cancer undergo a sentinel lymph node biopsy before a lumpectomy. The client asks the nurse the reason for removing the sentinel lymph node. Which of the following statements would the nurse state?

"It will prevent lymphedema, which is a common side effect."

According to the American Cancer Society, a woman in her 20s or 30s with no symptoms or family history of breast cancer should have a clinical breast examination once every ____ years.

3

While performing a clinical breast examination on a client, the nurse inspects the breasts with the client in various positions. Place the following positions in their proper sequence. 1 Arms resting at the sides 2 Arms raised over the head 3 Hands placed on the hips 4 Hands on hips while standing

3: Hands placed on the hips 1: Arms resting at the sides 4: Hands on hips while standing 2: Arms raised over the head During the clinical breast examination, the nurse inspects the breasts first with the client sitting at the edge of the examination table with her arms resting at her sides. Next, the client places her hands on her hips, and then she raises her arms over her head. Lastly, the client stands and places her hands on her hips and leans forward

A client presents to the office, upset, after finding a small lump in her breast. The nurse tries to reassure her that the lump does not automatically indicate cancer based on the understanding that what percentage of breast lumps discovered are caused by benign breast disorders?

80% Many women believe that all lumps are cancerous; however, more than 80% of all lumps discovered are benign and need no treatment. Patience, support, and education the essential components of nursing care.

As part of discharge planning, the nurse refers a woman to Reach to Recovery. This group's primary purpose is to: A) Help support women who have undergone mastectomies B) Raise funds to support early breast cancer detection programs C) Provide all supplies needed after breast surgery for no cost D) Collect statistics for research for the American Cancer Society

A

A woman comes to the clinic and tells the nurse that she has read an article about certain foods that have anticancer properties and help boost the immune system. The nurse identifies A) Garlic B) Soybeans C) Milk D) Leeks E) Flax seeds

ABDE

A patient who came to the clinic after finding a mass in her breast is scheduled for a diagnostic breast biopsy. What is it important for the nurse to do?

Acknowledge the fear the patient is experiencing

A nurse is working at a cancer treatment center and is developing programs for specific ethnic groups related to breast cancer. The nurse would target which ethnic group because it has the highest breast cancer mortality rate in the United States?

African American Although the incidence of breast cancer is highest in Caucasians, African American women have a higher breast cancer mortality rate at every age and a lower survival rate than any other racial or ethnic group.

When preparing a teaching plan for a woman with mastitis, which of the following would the nurse include?

Applying warm compresses to the affected breast

A client arrives at the health care facility complaining of a lump that she felt during her breast self-examination. Upon diagnosis, the physician suspects fibroadenomas. Which of the following questions should the nurse ask when assessing the client? a. do you consume foods high in fat? b. are you lactating? c. are you taking oral contraceptives? d. do you smoke?

Are you lactating? fibroadenomas can be stimulated by external estrogen, progesterone, lactation and pregnancy

While performing a clinical breast examination on a client, the nurse inspects the breasts with the client in various positions. Place the following positions in their proper sequence.

Hands placed on the hips Arms resting at the sides Hands on hips while standing Arms raised over the head

A 42-year-old woman is scheduled for a mammogram. Which of the following would the nurse include when teaching the woman about the procedure? A) "The room will be darkened throughout the procedure." B) "Each breast will be firmly compressed between two plates." C) "Make sure to refrain from eating or drinking after midnight." D) "A small needle will be inserted to get a sample for evaluation."

B

After teaching a woman how to perform breast self-examination, which statement would indicate that the nurse's instructions were successful? A) "I should lie down with my arms at my side when looking at my breasts." B) "I should use the fingerpads of my three middle fingers to apply pressure to my breast." C) "I don't need to check under my arm on that side if my breast feels fine." D) "I need to work from the center of my breast outward toward my shoulder."

B

The nurse is developing a plan of care for a woman with breast cancer who is scheduled to undergo breast-conserving surgery. The nurse interprets this as which of the following? A) Removal of nipple and areolar area B) Lump removal followed by radiation C) Entire breast removal without lymph nodes D) Axillary lymph node removal

B

Breast cancer that is localized is referred to as: a. Primary b. In situ c. Metastasized d. Localized

B" since it defines a small lesion that has not spread from its origin. Responses "A" and "C" and "D" are not terms used to describe a local cancerous lesion.

A biopsy procedure that traces radioisotopes and blue dye from the tumor site through the lymphatic system into the axillary nodes is: a. Stereotactic biopsy b. Sentinel node biopsy c. Axillary dissection biopsy d. Advanced breast biopsy

B. This describes the procedure for performing a sentinel node biopsy.

During a wellness visit to the clinic, a woman asks the nurse if there is anything she can do to reduce her risk for developing breast cancer. Which of the following would the nurse most likely include? (Select all that apply.) A) Eating three servings of fruit daily B) Keeping weight gain under 11 pounds after age 18 C) Eating at least seven portions of complex carbohydrates daily D) Limiting the intake of refined sugar products E) Using salt liberally when cooking

BCD

A young client desires to know her risk of developing breast carcinoma. She has a positive family history of breast carcinoma. The nurse would suggest that she should undergo which diagnostic test?

BRCA-1 genetic marker Individuals with BRCA1 and BRCA2 mutations have a 75% lifetime risk of breast cancer and a 30% lifetime risk of ovarian cancer. The genetic influences of BRCA1 and BRCA2 are recognized to be a risk factor for developing breast carcinoma among family members. DNA ploidy status is used to confirm a malignant breast mass. Digital mammography is used to screen for breast masses. A fine-needle biopsy is used to identify if a mass is benign or malignant.

A woman has been treated for a tumor of the left breast that has not responded to chemotherapy. The woman has just found out that she has the BRCA mutations and discusses her options with her physician. What treatment would be most difficult for this woman?

Bilateral mastectomy

After teaching a group of students about the signs and symptoms of breast cancer, the instructor determines that additional teaching is needed when the group identifies which of the following?

Breast symmetry

The nurse is developing the discharge plan for a woman who has had a left-sided modified radical mastectomy. The nurse is including instructions for ways to minimize lymphedema. Which suggestion would most likely increase the woman's symptoms? A) "Wear gloves when you are doing any gardening." B) "Have your blood pressure taken in your right arm." C) "Wear clothing with elasticized sleeves." D) "Avoid driving to and from work every day."

C

A group of students are reviewing information about benign and malignant breast masses. The students demonstrate understanding when they identify which of the following as indicating a benign breast mass. (Select all that apply.) A) Painless B) Unilateral location C) Firm consistency D) Absence of dimpling E) Fixed to chest wall

C D

A 25-year-old woman presents with an asymptomatic breast mass. Which of the following is true concerning her diagnosis and treatment? a. All breast masses should be considered premalignant b. The breast mass should be surgically removed immediately c. Ultrasound is typically used to determine the diagnosis d. Since it is asymptomatic, just reassurance is needed now

C" since an ultrasound can differentiate a solid mass from a cyst. In addition, the ultrasound appearance of a well-rounded, homogenous mass may be sufficiently characteristic to diagnose fibroadenoma. Response "A" is incorrect because up to 80% of breast masses are benign, especially in young women.

Breast self-examinations involve both touching of breast tissue and: a. Palpation of cervical lymph nodes b. Firm squeezing of both breast nipples c. Visualizing both breasts for any change d. A mammogram to evaluate breast tissue

C. Visible changes to the skin of the breast take place (dimpling, contour changes, nipple discharge) and can be seen if inspected in front of a mirror.

Mammography is recommended for a client diagnosed with intraductal papilloma. Which of the following factors should the nurse ensure when preparing the client for a mammography?

Client has not applied deodorant on the day of testing.

Mammography is recommended for a client diagnosed with intraductal papilloma. Which factor should the nurse ensure when preparing the client for a mammography?

Client has not applied deodorant on the day of testing. When preparing a client for mammography, the nurse should ensure the client has not applied deodorant or powder on the day of testing because these products can appear on the X-ray film as calcium spots. It is not necessary for the client to avoid fluid intake 1 hour prior to testing. Mammography has to be scheduled just after the client's menses to reduce chances of breast tenderness, not when the client is going to start her menses. The client can take aspirin or acetaminophen after the completion of the procedure to ease any discomfort, but these medications are not taken before mammography.

A female client with a malignant tumor of the breast has to undergo chemotherapy for a period of 6 months. Which of the following side effects should the nurse monitor for when caring for this client?

Constipation

A nurse is caring for a client who has just had her intraductal papilloma removed through a surgical procedure. What instructions should the nurse give this client as part of her care?

Continue monthly breast self-examinations.

A nurse is caring for a female client undergoing radiation therapy after her breast surgery. The client is refusing to eat and states she does not have a desire to eat at this time. The nurse would do which of the following first?

Continue to monitor the client

A 41-year-old female client arrives at a health care setting complaining of dull nipple pain with a burning sensation, accompanied by pruritus around the nipple. The physician suspects mammary duct ectasia. Which of the following orders should the nurse question?

Cool compresses to the affected areas (because it should be warm compresses)

A nurse is conducting a class on breast cancer prevention. Which statement would the nurse most likely include in the discussion? A) "Most often a lump is felt before it is seen." B) "Early breast cancer usually has some symptoms." C) "If the mass is not painful, it is usually benign." D) "If lump is palpable, it has been there for some time."

D

A 52-year-old female client with an estrogen receptors positive (ER+) breast cancer is undergoing hormonal therapy. While taking a selective estrogen receptor modulator (SERM), the client begins to experience hot flashes. What should the nurse do next?

Document the hot flash in the patient's chart

While interviewing a woman before her yearly examination, the nurse learns that she stopped breastfeeding over a year ago but occasionally has pain on one side behind her left nipple not related to her period. Upon examination, the nurse does not palpate any lumps but finds the left nipple retracted with a greenish discharge. What would the nurse first suspect?

Duct ectasia

A client has undergone a mastectomy for breast cancer. Which of the following instructions should the nurse include in the postsurgery client-teaching plan?

Elevate the affected arm on a pillow

Which of the following would the nurse include in the plan of care for a client diagnosed with breast cancer who has undergone a mastectomy?

Encouraging the client to perform arm exercises

What is the most common breast mass in women?

Fibroadenoma

Edema and enlargement in the ducts and glands of the breast that occur as effects of hormones are called

Fibrocystic breast changes

When reviewing a woman's history, the nurse identifies which factor as increasing the client's risk for breast cancer?

History of ovarian cancer

A 62-year-old female client arrives at a health care facility complaining of skin redness in the breast area, along with skin edema. The physician suspects inflammatory breast cancer. Which of the following is a symptom of inflammatory breast cancer that the nurse should assess for?

Increased warmth of the breast

A woman who is 6 months postpartum calls the clinic and reports flu-like symptoms, an elevated temperature, and pain and redness throughout her left breast. What would the nurse first suspect?

Mastitis

A client diagnosed with fibroadenoma is worried about her chances of developing breast cancer. She also asks the nurse about various breast disorders and their risks. Which of the following benign breast disorders should the nurse include as having the greatest risk for the development of breast cancer?

Intraductal papilloma

The nurse is developing a presentation for a local women's health center about breast cancer. Which of the following would the nurse include as being the most common type of breast cancer?

Invasive ductal carcinoma

What is the most common type of breast cancer, accounting for approximately 85% of cases?

Invasive ductal carcinoma

A woman is crying because she just recently received the results of her biopsies, and they confirm that she has invasive breast cancer. Which response by the nurse is the most appropriate?

Listen to the woman talk and remain silent for a while

The nurse is providing care to a client who has had surgery as treatment for breast cancer. The nurse would be alert for the development of which of the following?

Lymphedema

A nurse is preparing a presentation for a health fair about preventing breast cancer. Which of the following would the nurse include?

Maintain an ideal weight

A client is considering breast augmentation. Which of the following would the nurse recommend to the client to ensure that there are no malignancies?

Mammogram

Which of the following is TRUE regarding the treatment of breast cancer?

Often, the first treatment option for breast cancer is surgery.

During a recent visit to the clinic a woman presents with erythema of the nipple and areola on the right breast. She states this started several weeks ago and she was fearful of what would be found. The nurse suspects

Paget's Disease

Which breast disorder is NOT benign?

Paget's disease

A client complains of lumpy, tender breasts, particularly during the week before menses. She complains of pain that often dissipates after the onset of menses. The nurse suspects the client has fibrocystic breast changes. Which of the following should the nurse do next? a. determine if the client has had a mammography b. have the client follow up in a week c. breast examination d. schedule the client for cryoablation

Perform a breast examination. to determine is the client is experiencing fibrocystic breast

A woman is scheduled to undergo a modified radical mastectomy. Which of the following would the nurse include when describing this surgery to the client?

Removal of breast tissue, axillary nodes, and some chest muscles

Which of the following would the nurse expect to find when assessing the breasts of a client with fibrocystic breast disease?

Soft mass

A small amount of breast milk is obtained for culture and sensitivity testing from a client with mastitis. The nurse would expect the results to identify which organism as the most likely cause?

Staphylococcus aureus

A breast tumor is most likely found in what part of the breast?

The upper outer quadrant

A client with breast cancer is scheduled to undergo chemotherapy with aromatase inhibitors. Which of the following best reflects the rationale for using this group of drugs?

They lower the level of estrogen in the body blocking the tumor's ability to use it

A client complains of having tender and painful breasts, often feeling multiple lumps within her breast tissue. The nurse would need to gather additional information about which of the following?

Timing of symptoms in relation to the menstrual cycle

An OB/GYN nurse is teaching a new female client how to perform breast self-examination. Which of the following is a recommended step for this examination?

To examine the right breast, put a pillow or folded towel under the right shoulder

A client with breast cancer is to receive immunotherapy. The nurse would expect the client to receive which of the following?

Trastuzumab

During a follow-up visit, a female client who underwent a mastectomy asks the nurse if she can work in her backyard or at least do some household work. Which suggestion would be most appropriate?

Wear gloves and protective clothing to avoid any injuries

A client with a fibroadenoma is being scheduled for diagnostic testing. Which of the following would the nurse expect as most likely?

Ultrasound

In which client would the nurse suspect cancer?

a 45-year-old with thickening in one breast with nipple irritation and retraction and a pink discharge The 25-year-old most likely has fibrocystic breast changes. The 30-year-old most likely has a fibroadenoma. The 40-year-old most likely has an intraductal papilloma. Further assessment is needed to confirm each of these, but this is what the nurse would first suspect

What does the American Cancer Society (ACS) say about breast self-examination (BSE)?

a monthly BSE is optional

Which factors contribute to the development of breast cancer? a) Aging b) High breast density c) Family history d) Antibiotic use e) Hormonal factors

a, b, c, e: The following factors are thought to contribute to the development of breast cancer: aging, hormonal factors, delayed childbearing or not having children, high breast density, a family history of breast or ovarian cancer, and late menopause. Antibiotic use has not been identified as a factor contributing to the development of breast cancer.

A woman's breast examination reveals a small mass. Which of the following characteristics would lead the nurse to suspect that the mass is malignant? Select all that apply. a) Skin dimpling b) Hard on palpation c) Painless d) Clearly delineated e) Rubbery feel f) Mobile

a, b, c: Malignant breast masses are typically described as hard on palpation and painless, with skin dimpling. Benign breast masses are described as mobile and clearly delineated, with a rubbery feel

Lumpectomy is a treatment option for clients diagnosed with breast cancer with tumors smaller than 5 cm. For which clients is lumpectomy contraindicated? Select all that apply. a) Client who has had previous radiation to the affected breast b) Client whose surgery will not result in a clean margin of tissue c) Client who has had an early menarche or late onset of menopause d) Client whose connective tissue is reported to be sensitive to radiation e) Client who has failed to breastfeed for up to 1 year after pregnancy

a, b, d: Lumpectomy is contraindicated for women who have previously undergone radiation to the affected breast, those whose connective tissue is reported to be sensitive to radiation, and those whose surgery will not result in a clean margin of tissue. Clients who have had an early menarche or late onset of menopause and clients who have failed to breastfeed for up to 1 year after pregnancy are at risk for developing breast cancer. Lumpectomy is a treatment option for clients with breast cancer

A nurse is screening women for risk factors for breast cancer. Which of the following are considered a risk for this disorder? Select all that apply. a) Cystic breast disorders b) Birth of a child before age 25 c) Having breast-fed children d) Having an extreme fear of cancer e) Previous cancer f) Using female hormone therapy

a, d, e, f: Routine mammography is strongly recommended for women who have any of the following characteristics: previous cancer, cystic breast disorders, no children or birth of first child after age 30 years, no breast-fed children, family history of breast cancer, strong family history of any type of cancer, female hormone (estrogen) therapy, or extreme fear of cancer (need mammography for reassurance

A 33-year-old female client reports yellow nipple discharge and a pain in her breasts a week before menses that dissipates on the onset of menses. Diagnosis reveals that the client is experiencing fibrocystic breast changes. Which instructions should the nurse offer the client to help alleviate the condition? Select all that apply. a) Avoid caffeine b) Practice good hand-washing techniques c) Increase fluid intake steadily d) Take diuretics as recommended e) Maintain a low-fat diet

a, d, e: The nurse should instruct the client with fibrocystic breast changes to avoid caffeine. Caffeine acts as a stimulant that can lead to discomfort. It is important to maintain a lowfat diet rich in fruits, vegetables, and grains to maintain a healthy body weight. Taking diuretics is important to counteract fluid retention and swelling of the breasts. Practicing good handwashing techniques and increasing fluid intake are important for clients with mastitis but may not help clients with fibrocystic breast changes.

Which of the following are modifiable risk factors for breast cancer? Select all that apply. a) Sedentary lifestyle b) Smoking c) Duration of breastfeeding d) Alcohol consumption e) Obesity

a,b,c,d,e: All of the above are modifiable risk factors for breast cancer. Women who are nonsmokers, have a normal BMI, exercise, and drink little alcohol have a lower lifetime risk of breast cancer. Breastfeeding is inversely correlated with breast cancer. The more a woman breastfeeds, the less likely she is to be diagnosed with breast cancer

In the United States, what type of cancer accounts for one-third of cancer diagnoses and is the most common cancer in women? a) Breast cancer b) Uterine cancer c) Lung cancer d) Ovarian cancer

a: According to the American Cancer Society (ACS), in the United States, breast cancer is the most common cancer in women and accounts for 1 in 3 cancer diagnoses.

After teaching a woman about breast self-examination, which statement indicates that the teaching was successful? a) "I'll do the check about a week after my period." b) "I will check my breasts every other week." c) "I'll feel my breasts using my thumb and index finger." d) "I don't have to check under my arm if I don't feel any breast lumps."

a: Breast self-examination is best performed a week after menses, when swelling has subsided. Breast self-examination is typically performed every month. Both the breast area and the area between the breast and underarm, including the underarm itself, should be part of breast self-examination. The woman should use the pads of her three middle fingers for palpation

Edema and enlargement in the ducts and glands of the breast that occur as effects of hormones are called a) Fibrocystic breast changes b) Fibroadenomas c) Duct ectasia d) Intraductal papilloma

a: Changes in the breast that result from hormonal influences are fibrocystic breast changes.

A woman has been treated for a tumor of the left breast that has not responded to chemotherapy. The woman has just found out that she has the BRCA mutations and discusses her options with her physician. What treatment would be most difficult for this woman? a) Bilateral mastectomy b) Left mastectomy c) Radiation therapy d) More aggressive chemotherapy

a: Chemotherapy, left mastectomy, and radiation therapy may be difficult for the woman, but the most difficult and controversial treatment is bilateral mastectomy. Right mastectomy is considered a prophylactic mastectomy which is a primary prevention modality. Patients who are considering prophylactic mastectomy are often faced with a very controversial and emotion decision

A patient who came to the clinic after finding a mass in her breast is scheduled for a diagnostic breast biopsy. What is it important for the nurse to do? a) Acknowledge the fear the patient is experiencing b) Set up a consultation with a breast cancer survivor c) Instruct the patient on prosthetic devices d) Refer her to Reach for Recovery

a: In the breast cancer diagnostic phase it is appropriate to acknowledge her feelings of fear, concern, and apprehension. Referral to Reach for Recovery and with a breast cancer survivor is indicated when the diagnosis has been made and surgical intervention is scheduled. Instructing the patient on prosthetic devices is only appropriate when the patient has had or is scheduled for a mastectomy

During a routine physical examination, a firm mass is palpated in the right breast of a 35-year-old woman. Which finding or client history would suggest cancer of the breast as opposed to fibrocystic disease? a) Increased vascularity of the breast b) Mass located in upper, outer quadrant c) Cyclic change in mass size d) History of anovulatory cycles

a: Increase in breast size or vascularity is consistent with breast cancer. Masses associated with fibrocystic disease of the breast are firm, are most commonly located in the upper outer quadrant of the breast, and increase in size before menstruation. They may be bilateral in a mirror image and are typically well demarcated and freely moveable

A small amount of breast milk is obtained for culture and sensitivity testing from a client with mastitis. The nurse would expect the results to identify which organism as the most likely cause? a) Staphylococcus aureus b) Group A streptococcus c) Chlamydia trachomatis d) Escherichia coli

a: The most common causative microorganism associated with mastitis is Staphylococcus aureus. Chlamydia is a sexually transmitted infection. Streptococcus is commonly associated with strep throat. E. coli is a common cause of urinary tract infections

A client diagnosed with fibroadenoma is worried about the chances of developing breast cancer. She also asks the nurse about various breast disorders and their risks. Which benign breast disorder should the nurse include as having the greatest risk for the development of breast cancer? a) Intraductal papilloma b) Fibrodenomas c) Mastitis d) Mammary duct ectasia

a: The nurse should inform the client that intraductal papillomas and fibrocystic breasts, although considered benign, carry a cancer risk with prolific masses and hyperplastic changes within the breasts. Other benign breast disorders such as mastitis, mammary duct ectasia, and fibroadenomas carry little risk.

It is recommended that a 48-year-old female client with breast cancer undergo a sentinel lymph node biopsy before a lumpectomy. The client asks the nurse the reason for removing the sentinel lymph node. Which statement will the nurse make? a) "It will prevent lymphedema, which is a common side effect." b) "It will decrease the amount of treatment you need." c) "It will reveal the hormone receptor status of the cancer." d) "It will lessen the aggressiveness of the subsequent chemotherapy."

a: The nurse should inform the client that removing only the sentinel lymph node prevents side effects such as lymphedema, which is otherwise associated with a traditional axillary lymph node dissection. It does not help reveal the hormonal status of the cancer. Hormone-receptor status can be revealed through normal breast epithelium, which has hormone receptors and responds specifically to the stimulatory effects of estrogen and progesterone. A sentinel lymph node biopsy will determine how powerful a chemotherapy regimen the client will have to undergo, but undergoing a sentinel lymph node biopsy will not lessen the aggressiveness of the chemotherapy. Degree of HER-2/neu oncoprotein will be revealed through the HER-2/neu genetic marker, not through a sentinel lymph node biopsy

A female client with a malignant tumor of the breast has to undergo chemotherapy for a period of 6 months. For which side effect should the nurse monitor when caring for this client? a) Constipation b) Chills c) Vaginal discharge d) Headache

a: The side effects of chemotherapy are constipation, hair loss, weight loss, vomiting, diarrhea, immunosuppression, and, in extreme cases, bone marrow suppression. The nurse should monitor for these side effects when caring for the client undergoing chemotherapy. Vaginal discharge, headache, and chills are not side effects of chemotherapy. Vaginal discharge is one of the side effects of SERMs as a part of hormonal therapy, which is used to prevent cancer from spreading further into the body. Headache is a side effect of aromatase inhibitors under hormonal therapy to counter cancer. Chills are a side effect of immunotherapy

When preparing a teaching plan for a woman with mastitis, which instruction would the nurse include? a) Applying warm compresses to the affected breast b) Ceasing breastfeeding to prevent transmission to the infant c) Limiting fluid intake to minimize swelling d) Avoiding the use of a supportive bra to prevent pressure on the inflamed breast

a: Warm compresses are soothing and help reduce inflammation. Breastfeeding is encouraged with mastitis to ensure continued emptying of the breast. Fluid intake is encouraged to promote milk production and resolution of infection. Wearing a supportive bra 24 hours a day is necessary to support the breasts

A woman is crying because she just recently received the results of her biopsies, and they confirm that she has invasive breast cancer. Which response by the nurse is the most appropriate? a) Listen to the woman talk and remain silent for a while b) "You'll beat this thing, I know it. You are very strong." c) "I'm sure you are going to be fine. You are in great hands." d) "I know a great support group you can join."

a: When a woman first receives the devastating news of the diagnosis of cancer, most often the best response is to allow the woman to express her feelings and concerns before speaking. Giving her false reassurances is not therapeutic and can break reliability and trust in a provider/patient relationship. Attempting to give her information about groups or next steps before she is in a state to take it in is also nontherapeutic

A nurse is caring for a client who has just had her intraductal papilloma removed through a surgical procedure. What instructions should the nurse give this client as part of her care? a) Continue monthly breast self-examinations b) Wear a supportive bra 24 hours per day c) Refrain from consuming salt in diet d) Apply warm compresses to the affected breast

a: When caring for a client who has just undergone surgery for intraductal papilloma, the nurse should instruct the client to continue monthly breast self-examinations along with yearly clinical breast examinations. Applying warm compresses to the affected breast and wearing a supportive bra 24 hours a day are instructions given in cases of mastitis but not for intraductal papilloma. The nurse should instruct clients to refrain from consuming salt in the diet in cases of fibrocystic breast changes but not in cases of intraductal papilloma

A 52-year-old female client with an estrogen receptors positive (ER+) breast cancer is undergoing hormonal therapy. While taking a selective estrogen receptor modulator (SERM), the client begins to experience hot flashes. What should the nurse do next? a) Document the hot flash in the client's chart b) Notify the client's health care provider c) Instruct the client to stop taking the SERM d) Assess the client's blood pressure

a: When caring for a client who is being administered selective estrogen receptor modulator, the nurse should monitor for side effects such as hot flashes, vaginal discharge, bleeding, and cataract formation. Hot flashes are an expected side effect of SERM; therefore the nurse should document the finding in the chart

Mammography is recommended for a client diagnosed with intraductal papilloma. Which factor should the nurse ensure when preparing the client for a mammography? a) Client has not applied deodorant on the day of testing. b) Client is just going to start her menses. c) Client has taken an aspirin before the testing. d) Client has not consumed fluids 1 hour before testing.

a: When preparing a client for mammography, the nurse should ensure the client has not applied deodorant or powder on the day of testing because these products can appear on the x-ray film as calcium spots. It is not necessary for the client to avoid fluid intake 1 hour prior to testing. Mammography has to be scheduled just after the client's menses to reduce chances of breast tenderness, not when the client is going to start her menses. The client can take aspirin or acetaminophen after the completion of the procedure to ease any discomfort, but these medications are not taken before mammography

A 38-year-old female client has to undergo lymph node surgery in conjunction with mastectomy. The client is likely to experience lymphedema due to the surgery. Postsurgery, which factors will make the client more susceptible to lymphedema? Select all that apply. a) Not consuming a diet high in fiber and protein b) Not wearing a well-fitted compression sleeve c) Using the affected arm for drawing blood or measuring blood pressure d) Engaging in activities like gardening without using gloves e) Not consuming foods that are rich in phytochemicals

b, c, d: The client is more susceptible to lymphedema if the affected arm is used for drawing blood or measuring blood pressure, if she engages in activities like gardening without using gloves, or if she's not wearing a well-fitted compression sleeve to promote drainage return. Consuming foods rich in phytochemicals is essential to prevent the incidence of cancer, not lymphedema. Not consuming a diet high in fiber and protein will not make the client susceptible to lymphedema

A nurse is educating a 43-year-old female client about required lifestyle changes to help avoid breast cancer. Which instructions regarding diet and food habits should the nurse include in the teaching plan? Select all that apply. a) Consume at least five servings of proteins daily b) Limit intake of processed foods c) Restrict intake of salted foods d) Consume seven or more portions of complex carbohydrates daily e) Increase liquid intake to 3 L daily

b, c, d: The nurse should instruct the client to restrict intake of salted foods, limit intake of processed foods, and consume seven or more daily portions

How should the nurse instruct a woman to perform a breast self-examination? Select all that apply. a) "Use two different degrees of pressure: light for the skin and hard for the bone." b) "You will need to visually inspect both breasts and palpate them for changes." c) "Use the pads of your first two fingers on the right hand for the right breast." d) "Look for any changes in shape, size, contour, or symmetry between the breasts, as well as any skin discoloration." e) "It doesn't matter which pattern you use to palpate the breast as long as you cover the entire breast."

b, d, e: To palpate the breasts, the woman should use the pads of three fingers from the opposite hand to the opposite breast (left to right). The nurse should instruct the woman to do three different levels of pressure, light (skin and tissue), medium (tissue), and hard (bone).

When counseling a woman with monthly breast pain, what dietary recommendations should the nurse provide? a) "Caffeine may help with monthly breast pain." b) "Eat plenty of fruits, vegetables, and whole grains, and follow a low-fat diet." c) "Drink milk and eat yogurt." d) "Be sure to consume plenty of electrolytes, including salt."

b: Maintaining a healthy weight will reduce pain from fibrocystic breasts. Caffeine and too much salt can contribute to fibrocystic breast changes. Calcium does not have a conclusive effect on breast pain

The nurse is reviewing the history and physical exam of a woman who has come to the clinic for a routine physical. Which factor would the nurse identify as increasing the client's risk for breast cancer? a) Asian race b) History of ovarian cancer c) Menarche at age 14 d) 39 years of age

b: A personal history of ovarian cancer is considered a risk factor for breast cancer. Typically, breast cancer is associated with aging (women over 50 years of age). Breast cancer is more common in Caucasian women, but African-American women are more likely to die of it. Early menarche (before 12 years of age) or late onset of menopause (after age 55 years) is associated with an increased risk for breast cancer

A client complains of having tender and painful breasts, often feeling multiple lumps within her breast tissue. The nurse would need to gather additional information about which of the following? a) Client's workplace in relation to the surroundings b) Timing of symptoms in relation to the menstrual cycle c) Bathing frequency and living surroundings d) Alcohol and caffeine consumption

b: Considering that the client has tender and painful breasts and that she often feels lumps within her breast tissue, it is most likely that she suffers from fibrocystic breast disease. To confirm these findings, the nurse should ask relevant questions about the characteristics and timing of symptoms in relation to the menstrual cycle. Symptoms of fibrocystic breast disease are noticeable before menstruation and usually abate during menstruation. The size of the cyst becomes larger before menstruation and often changes with the menstrual cycle. The nurse should further ask the client about her habits of smoking and consuming coffee, chocolate, and caffeinated soft drinks, not alcohol, because they aggravate the condition. Workplace surroundings or cleanliness habits do not matter because fibrocystic breast disease is not infectious

A client complains of having tender and painful breasts, often feeling multiple lumps within her breast tissue. The nurse would need to gather additional information about which of the following? a) Alcohol and caffeine consumption b) Timing of symptoms in relation to the menstrual cycle c) Bathing frequency and living surroundings d) Client's workplace in relation to the surroundings

b: Considering that the client has tender and painful breasts and that she often feels lumps within her breast tissue, it is most likely that she suffers from fibrocystic breast disease. To confirm these findings, the nurse should ask relevant questions about the characteristics and timing of symptoms in relation to the menstrual cycle. Symptoms of fibrocystic breast disease are noticeable before menstruation and usually abate during menstruation. The size of the cyst becomes larger before menstruation and often changes with the menstrual cycle. The nurse should further ask the client about her habits of smoking and consuming coffee, chocolate, and caffeinated soft drinks, not alcohol, because they aggravate the condition. Workplace surroundings or cleanliness habits do not matter because fibrocystic breast disease is not infectious.

Which of the following would the nurse include in the plan of care for a client diagnosed with breast cancer who has undergone a mastectomy? a) Instructing client to use arm pumps b) Encouraging the client to perform arm exercises c) Applying ace bandages to the arms d) Counseling the client to apply elastic sleeves to her arms

b: Encouraging the client to perform arm exercises is important to maintain circulation to the arm on the affected side. Use of elastic sleeves, arm pumps, and ace bandaging is recommended for clients who may have complications following radiation or surgery for lymphedema of the arm

A 45-year-old woman comes into the OB/GYN clinic for her yearly check-up. The woman mentions to the nurse that she has a dimpling of the right breast that has occurred in the last 2 weeks. She has not performed a self-breast examination. What assessment would be appropriate for the nurse to make? a) Order an immediate mammogram b) Palpate the area for a breast mass c) Call the physician to schedule a biopsy d) Evaluate the patient's milk production

b: It would be most important for the nurse to palpate the breast to determine the presence of a mass. Edema and pitting of the skin may result from a neoplasm blocking lymphatic drainage, giving the skin an orange-peel appearance (peau d'orange), a classic sign of advanced breast cancer. Evaluation of milk production is required in lactating women. There is no indication of lactation in the scenario. A mammogram is an appropriate diagnostic test, but unless the nurse is a practitioner this would not be within the nurse's realm of practice. A referral could be made to a physician but the nurse would not proceed to schedule a biopsy

All of the following are characteristics of malignant breast disease EXCEPT a) Immobile, fixed masses b) Smooth, firm, rubbery masses c) Skin dimpling and nipple retraction d) Painless and nontender masses

b: Malignant breast disease is generally characterized by immobile, fixed lumps that may cause skin dimpling and nipple retraction. They are generally painless and not tender. They usually are not smooth and have poorly delineated edges

Which of the following pharmacologic treatments would NOT be therapeutic for a woman with fibrocystic breast changes? a) Diuretics b) Antihistamines c) Oral contraceptives d) Anti-inflammatory agents

b: Oral contraceptives will keep hormone levels stable to avoid peaks and troughs, which may cause breast edema and pain. Anti-inflammatory medications will be helpful in reducing pain from fibrocystic breast changes. Diuretics can be helpful in reducing excess fluid leading to edema and subsequent pain in the breast. Antihistamines are used to treat an overactive immune response, not edema from breast changes

You are caring for a 52-year-old woman whose sisters and mother died of breast cancer. The patient states, "My doctor wants me to take tamoxifen to help prevent breast cancer. What do you think?" What would be your best response? a) "Tamoxifen prevents osteoporosis." b) "Tamoxifen reduces the incidence." c) "I would recommend raloxifene." d) "We call this drug a chemotherapy agent."

b: Tamoxifen is referred to as a chemopreventive agent, not a chemotherapeutic agent. It does prevent osteoporosis, but this response does not address breast cancer prevention. Raloxifene (Evista) is another drug that shows promise as a chemopreventive agent. Tamoxifen reduces breast cancer incidence by 49%

Which of the following would the nurse expect to find when assessing the breasts of a client with fibrocystic breast disease? a) Nipple retraction b) Soft mass c) Skin dimpling d) Enlarged lymph nodes

b: The characteristic breast mass of fibrocystic disease is soft to firm, movable, and unlikely to cause nipple retraction. Nipple retraction, enlarged lymph nodes, and skin dimpling are more commonly associated with breast cancer

A female client who has a 2-month-old infant arrives at a health care facility reporting flu-like symptoms with fever and chills. When examining the breast, the nurse observes an increase in warmth. Which instruction should the nurse provide the client to help her cope with the condition? a) Avoid changing positions while nursing b) Increase fluid intake c) Avoid breastfeeding for 1 month d) Apply cold compresses to the affected breast

b: The nurse should instruct the client with mastitis to increase her fluid intake. A client with mastitis is instructed to continue breastfeeding as tolerated and to frequently change positions while nursing. The nurse should also instruct the client to apply warm, not cold, compresses to the affected breast area or to take a warm shower before breastfeeding

A 41-year-old female client arrives at a health care setting reporting dull nipple pain with a burning sensation, accompanied by pruritus around the nipple. The physician suspects mammary duct ectasia. Which order should the nurse question? a) Penicillin orally for 10 days b) Cool compresses to the affected area c) Acetaminophen as needed for discomfort d) Monitor temperature

b: The symptom of mammary duct ectasia include the presence of green, brown, straw-colored, reddish, gray, or cream-colored nipple discharge with a consistency of toothpaste. Treatment includes antibiotic therapy with penicillinase-resistant penicillin or cephalosporin, pain medication, and warm compresses to the inflamed area

A client with a fibroadenoma is being scheduled for diagnostic testing. Which of the following would the nurse expect as most likely? a) Excisional biopsy b) Ultrasound c) Mammogram d) Culture of discharge

b: Ultrasound can reveal physical characteristics unique to a fibroadenoma versus malignant mass with a higher degree of accuracy than mammography. In the case of very young women—an atypical age for breast cancer—an excisional biopsy is performed only if the mass changes or becomes larger. If the mass is detected in a woman with a higher risk for developing breast cancer, such as one with a family history or of an older age, a biopsy is performed to confirm that the tissue is indeed benign. There is no discharge to culture

A client is considering breast augmentation. Which of the following would the nurse recommend to the client to ensure that there are no malignancies? a) Mastopexy b) Mammogram c) Breast biopsy d) Ultrasound

b: When caring for a client considering breast augmentation, the nurse should provide her with a general guideline to have a mammogram to verify that there are no malignancies. Mastopexy involves a breast life for drooping breasts. Ultrasound or breast biopsy would not be necessary unless there was evidence of a problem

A nurse is educating a client on the technique for performing breast self-examination. Which instruction should the nurse include in the teaching plan with regard to the different degrees of pressure that need to be applied on the breast? a) Light pressure midway into the tissue b) Hard pressure applied down to the ribs c) Medium pressure on the skin throughout d) Medium pressure around the areolar area

b: When performing the breast self-examination, the nurse should instruct the client to apply hard pressure down to the ribs. Light, not medium, pressure should be applied when moving the skin without moving the tissue underneath. Medium, not light, pressure should be applied midway into the tissue. Client need not specifically palpate the areolar area during breast self-examination

Mammography is recommended for a client diagnosed with intraductal papilloma. Which factor should the nurse ensure when preparing the client for a mammography? a) Client has taken an aspirin before the testing. b) Client has not applied deodorant on the day of testing. c) Client is just going to start her menses. d) Client has not consumed fluids 1 hour before testing.

b: When preparing a client for mammography, the nurse should ensure the client has not applied deodorant or powder on the day of testing because these products can appear on the x-ray film as calcium spots. It is not necessary for the client to avoid fluid intake 1 hour prior to testing. Mammography has to be scheduled just after the client's menses to reduce chances of breast tenderness, not when the client is going to start her menses. The client can take aspirin or acetaminophen after the completion of the procedure to ease any discomfort, but these medications are not taken before mammography.

A client has undergone a mastectomy for breast cancer. Which instruction should the nurse include in the postsurgery client-teaching plan? a) Restrict intake of medication b) Elevate the affected arm on a pillow c) Breathe rapidly for an hour d) Avoid moving the affected arm in any way

b: When providing care to the client, the nurse should instruct the client to elevate the affected arm on a pillow. As part of the respiratory care, the nurse should instruct the client to turn, cough, and breathe deeply every 2 hours; rapid breathing is not encouraged. Active range-of-motion and arm exercises are necessary. To counter any pain experienced by the client, analgesics are administered as needed; intake of medication is not restricted

Which of the following is TRUE regarding the treatment of breast cancer? a) Women with hormone-sensitive cancers often have a poor prognosis and require many different types of treatment. b) Often, the first treatment option for breast cancer is surgery. c) Immunotherapy is seldom used as an adjunct therapy. d) Because breast cancer is often a systemic disease, chemotherapy is not effective

b: Women with hormone-sensitive cancers have a good prognosis and often need only hormonal treatment. Immunotherapy is often used as an adjunct to surgery. Chemotherapy is effective in treating systemic cancers

A nurse is assigned to educate a group of women on cancer awareness. Which risk factors for breast cancer are modifiable? Select all that apply. a) Previous abnormal breast biopsy b) Early menarche or late menopause c) Failing to breastfeed for up to a year after pregnancy d) Postmenopausal use of estrogen and progestins e) Not having children until after age 30

c, d, e: The modifiable risk factors for breast cancer are postmenopausal use of estrogen and progestins, not having children until after the age of 30, and failing to breastfeed for up to a year after pregnancy. Early menarche or late menopause and previous abnormal breast biopsy are the nonmodifiable risk factors for breast cancer

What is the most common breast mass in women? a) Mastitis b) Duct ectasia c) Fibroadenoma d) Intraductal papilloma

c: A fibroadenoma is a benign mass in the breast. It is the most common breast mass among women.

A nurse is caring for a female client undergoing radiation therapy after her breast surgery. The client is refusing to eat and states she does not have a desire to eat at this time. Which action should the nurse do first? a) Assess the client's BMI b) Notify the health care provider c) Continue to monitor the client d) Begin parenteral nutrition

c: A nurse would monitor for signs of anorexia as it is a likely side effect of radiation therapy, along with swelling and heaviness of the breast, local edema, inflammation, and sunburn-like skin changes. The nurse would continue to monitor the client since this is a common, expected side effect of radiation

The nurse is teaching a client with mastitis about care measures. Which client statement indicates effective teaching? a) "I should avoid pumping my breasts until the infection is cleared." b) "I need to wear a breast shield to keep the area moist." c) "I can use warm soaks to my breast to relieve the discomfort." d) "I can stop the antibiotic after 5 days."

c: Applying warm soaks to the breast or letting warm water from the shower flow over the breast can help to relieve some of the discomfort. Breast shields should be avoided because they trap breast milk and moisture around the nipple. The client needs to continue antibiotic therapy as prescribed for the entire treatment period, usually 10 days. The client can express milk with a breast pump until the infection resolves sufficiently to resume breast feeding

A client with breast cancer is scheduled to undergo chemotherapy with aromatase inhibitors. Which of the following best reflects the rationale for using this group of drugs? a) They block progesterone dependent tumors from growing b) They stimulate the immune system to attack a protein common in many tumors c) They lower the level of estrogen in the body blocking the tumor's ability to use it d) They attach to endogenous protein receptors to slow the growth of cancerous cells

c: Aromatase inhibitors lower the level of estrogen in the body thereby interfering with the ability of hormone-sensitive tumors to use estrogen for growth. Antiprogestin drug such as mifepristone, blocks progesterone-dependent breast cancers. The monoclonal antibody, trastuzumab attaches to protein receptors to slow the growth of cancer cells. A breast cancer vaccine is under investigation in Italy. This vaccine stimulates the immune system to attack a protein called mammoglobin A, which is found in 80% of breast cancer tumors

A client is scheduled to have a screening mammogram. When teaching the woman about this test, which instruction would be most appropriate for the nurse to include? a) "You can leave your necklace on, but you'll need to take off your watch and earrings." b) "You'll need to take a narcotic analgesic for any discomfort afterwards." c) "Don't use deodorant that day because it may interfere with the x-ray image." d) "Schedule the procedure just before your menses."

c: Deodorants and powder can appear on the x-ray film as calcium spots. The procedure should be scheduled just after menses, when breast tenderness is reduced. All jewelry must be removed from around the woman's neck because metal can cause distortions on the film image. Acetaminophen or aspirin can relieve any discomfort after the procedure

What is the most common type of breast cancer, accounting for approximately 85% of cases? a) Tubular carcinoma b) Medullary carcinoma c) Invasive ductal carcinoma d) Invasive lobular carcinoma

c: Invasive ductal carcinoma is the most common type of breast cancer. Invasive lobular carcinoma is the second most common type of breast cancer and accounts for approximately 10% to 15% of all cases

A nursing diagnosis of Risk for impaired tissue integrity would be most appropriate for which client? a) Client with a vaginal packing in place b) Client with endometriosis c) Client having reconstructive breast surgery d) Client taking oral contraceptives

c: Reconstructive breast surgery places the client at risk for insufficient blood supply to the muscle graft and skin, which can lead to tissue necrosis. Endometriosis or oral contraceptives aren't generally associated with altered tissue perfusion. Pressure from vaginal packing can sometimes put pressure on the bladder neck and interfere with voiding

A 62-year-old female client arrives at a health care facility reporting skin redness in the breast area, along with skin edema. The physician suspects inflammatory breast cancer. For which symptom of inflammatory breast cancer should the nurse assess? a) Palpable mobile cysts b) Induced nipple discharge c) Increased warmth of the breast d) Palpable papilloma

c: Skin edema, redness, and warmth of the breast are symptoms of inflammatory breast cancer. Induced discharge is an indication of benign breast conditions, which are noncancerous. Cancer involves spontaneous nipple discharge. Papillomas and palpable mobile cysts are characteristics of fibroadenomas, intraductal papilloma, and mammary duct ectasia, which are benign breast conditions and are noncancerous

After teaching a client with mastitis about caring for her breasts, the nurse determines that the client has understood the instructions when she states which of the following? a) " Until the infection is gone, I can't take a tub bath or shower." b) "I should avoid wearing a bra because it would increase the pain." c) "I will express milk with a breast pump until infection is resolved." d) "I need to apply cold soaks to the breast several times a day."

c: The client statement about expressing milk with a breast pump until the infection is resolved indicates effective teaching. The client should apply warm soaks to the breast or let warm water from a shower flow over the breast. The nurse should advise the client to wear a supportive brassiere. The client should bathe or shower regularly

The nurse is developing a presentation for a local women's health center about breast cancer. Which of the following would the nurse include as being the most common type of breast cancer? a) Inflammatory breast cancer b) Medullary carcinoma c) Invasive ductal carcinoma d) Infiltrating ductal carcinoma

c: The most common malignancy is invasive ductal carcinoma (85%), followed by infiltrating ductal carcinoma (75%), tubular carcinoma, colloid carcinoma, medullary, and inflammatory breast cancer, the rarest but most aggressive form of breast cancer

During a follow-up visit, a female client who underwent a mastectomy asks the nurse if she can work in her backyard or at least do some household work. Which suggestion would be most appropriate? a) Increase the frequency of follow-up visits if she does works b) Avoid working in the garden or yard altogether c) Wear gloves and protective clothing to avoid any injuries d) Avoid household chores for at least 6 to 9 months

c: The nurse should recommend that the client wear gloves when doing backyard work or housework to prevent injuries that may heal slowly or become infected. Working whether it be in the backyard or doing some household chores can be helpful in promoting feelings of usefulness, thereby enhancing the client's coping abilities and self-esteem. She could be advised to follow up more frequently however, this would not help prevent any untoward injury

The nurse is giving an educational event for a local women's group on self-examination of the breast. The nurse tells the women's group to raise their arms and inspect their breasts in a mirror. A member of the women's group asks the nurse why she needs to do this. What is the nurse's best response? a) "If you feel pain you will need to inspect it." b) "It will give you greater visibility." c) "It will help to observe for dimpling." d) "Everyone is different in assessing the breast."

c: The primary reason for raising the arms is to detect any dimpling. To elicit skin dimpling or retraction that may otherwise go undetected, the examiner instructs the patient to raise both arms overhead. This maneuver normally elevates both breasts equally. Pain does not always occur with breast masses. The assessment of the breast should be uniform.

A woman who is 6 months postpartum calls the clinic and reports flu-like symptoms, an elevated temperature, and pain and redness throughout her left breast. What would the nurse first suspect? a) Duct ectasia b) Inflammatory breast cancer c) Mastitis d) Intraductal papilloma

c: These symptoms most closely resemble mastitis. Mastitis usually occurs in the postpartum period while the woman is still breastfeeding. Mastitis is usually unilateral and is seen as a red, painful breast with elevated temperature and flu-like symptoms

While performing a clinical breast examination, the nurse notes a firm and rubbery nodule that is well circumscribed and moves freely. How should the nurse counsel the patient? a) "This is a normal breast finding, and you don't have to worry about it." b) "You may have breast cancer." c) "It's most likely a fibroadenoma, but we may need to do a biopsy."

c: This description most closely matches a fibroadenoma, but diagnostic imaging and even biopsy are warranted to confirm and rule out a cancerous tumor. The nurse should never tell the patient that she may have cancer because this will only cause anxiety

An OB/GYN nurse is teaching a new female client how to perform breast self-examination. Which of the following is a recommended step for this examination? a) Standing up, press gently in small circular motions around an imaginary clock face b) In the shower, place hands on hips, bend forward and examine breasts c) To examine the right breast, put a pillow or folded towel under the right shoulder d) Before a mirror, inspect the breasts with arms raised, and then extended sideways

c: To examine the right breast, the client should put a pillow or folded towel under the right shoulder and place the right hand behind the head. With the left hand, fingers flat, press gently in small circular motions around an imaginary clock face. Before a mirror, the client should inspect the breast with arms at side, then raise hands above the head and inspect the breasts

What does the American Cancer Society (ACS) say about breast self-examination (BSE)? a) If a patient has mammography every year, she needs to perform a BSE only every 6 months. b) As long as a patient has a clinical breast examination every 1 to 3 years, a BSE is recommended only every 6 months. c) To decrease mortality from breast cancer, a BSE should be performed every month. d) A monthly BSE is optional.

d: According to the American Cancer Society, a monthly BSE is now optional because the most recent research demonstrates that monthly BSEs do not decrease the mortality associated with breast cancer

A female client with metastatic breast disease is receiving trastuzumab as part of her immunotherapy. The client has nausea, fatigue, diarrhea, appears jaundice, and has a distended abdomen. What would the nurse do next? a) Decrease the trastuzumab infusion rate b) Continue to monitor the client c) Assess the client's white blood cell count d) Notify the healthcare provider

d: Adverse effects of trastuzumab include cardiac toxicity, vascular thrombosis, hepatic failure, fever, chills, nausea, vomiting, and pain with first infusion. The nurse should monitor for these adverse effects with the first infusion of trastuzumab. The nurse would notify the health care provider since the client is showing signs of hepatic failure.

A breast tumor is most likely found in what part of the breast? a) At the areola b) Near the nipple c) The upper inner quadrant d) The upper outer quadrant

d: Although they can occur throughout the breast, breast tumors are most frequently located in the upper outer quadrant of the breast.

The nurse is teaching a client with mastitis about care measures. Which client statement indicates effective teaching? a) "I need to wear a breast shield to keep the area moist." b) "I should avoid pumping my breasts until the infection is cleared." c) "I can stop the antibiotic after 5 days." d) "I can use warm soaks to my breast to relieve the discomfort."

d: Applying warm soaks to the breast or letting warm water from the shower flow over the breast can help to relieve some of the discomfort. Breast shields should be avoided because they trap breast milk and moisture around the nipple. The client needs to continue antibiotic therapy as prescribed for the entire treatment period, usually 10 days. The client can express milk with a breast pump until the infection resolves sufficiently to resume breast feeding

A client arrives at the health care facility reporting a lump that she felt during her breast self-examination. Upon diagnosis, the physician suspects fibroadenomas. Which question should the nurse ask when assessing the client? a) "Do you consume foods high in fat?" b) "Do you smoke regularly?" c) "Are you taking oral contraceptives?" d) "Are you lactating?"

d: As the physician suspects fibroadenomas, it is important for the nurse to know whether the client is pregnant or lactating since the incidence of fibroadenomas is more frequent among pregnant and lactating women. Taking oral contraceptives assists a client with fibrocystic breast changes, but is not necessary for a client with fibroadenomas. Fibroadenomas usually occur in women between 20 and 30 years of age. Smoking and a high-fat diet will make the client more susceptible to cancer, not fibroadenomas.

The nurse is preparing a client for breast biopsy of a lump. The nurse explains that a needle with a cutting edge will be inserted into a lump and rotated to remove a core sample. What type of breast biopsy is this client undergoing? a) Aspiration b) Incisional biopsy c) Excisional biopsy d) Needle biopsy

d: During a needle biopsy, a needle with a cutting edge will be inserted into a lump and rotated to remove a core sample. With aspiration, cells from a lump are drawn into a syringe. In excisional biopsy, an entire lump is removed and analyzed, and, in incisional biopsy, part of a lump is removed as a sample

How should the nurse counsel a postpartum patient on how to prevent mastitis? a) "Be sure to keep your breasts covered when you are not feeding or pumping." b) "Sterilize your bottles and pump equipment after each use." c) "If you notice that your breast is warm, hard, or red, stop feeding on that side and pump from that breast instead." d) "Wash your hands thoroughly, and let your breasts dry after each feeding."

d: Handwashing is one of the best ways to prevent infection. If the woman feels that her breast is warm, hard, or red, she should increase the amount of breastfeeding from that side. It is not necessary to sterilize bottles and pumping equipment after each use. Normal dish washing is sufficient. Keeping the breasts exposed to the air to dry will aid in preventing infection

The nurse is providing care to a client who has had surgery as treatment for breast cancer. The nurse would be alert for the development of which of the following? a) Fibroadenoma b) Breast abscess c) Fibrocystic breast disease d) Lymphedema

d: Lymphedema occurs in some women after breast cancer surgery. It causes disfigurement and increases the lifetime potential for infection and poor healing. Fibrocystic breast disease and fibroadenoma are two benign breast conditions that occur usually in premenopausal woman. Breast abscess is the infectious and inflammatory breast condition that is common among breast-feeding mothers

A nurse is preparing a presentation for a health fair about preventing breast cancer. Which of the following would the nurse include? a) Opting for estrogen only replacement therapy during menopause b) Increasing the intake of caffeinated drinks c) Delaying childbearing until after age 30 d) Maintaining an ideal weight

d: Maintaining an ideal weight decreases the risk of breast cancer. Having no children or having children after age 30 is associated with an increased risk for breast cancer. Some breast tumors are hormone dependent, such that estrogen (or progesterone) enhances tumor growth. Women are advised to avoid the consumption of alcohol, not caffeine, because alcohol correlates with an increased risk of breast cancer

The nurse explains to a client with a family history of breast cancer the difference between benign and malignant neoplasms. Which of the following is a characteristic of a malignant neoplasm? a) Round or oval shape b) Movable c) Smooth border d) Irregular shape and hard

d: Malignant lesions are more likely to be irregularly shaped and hard, and often show secondary signs, such as enlarged lymph nodes in the axillary area, breast asymmetry, nipple retraction, bloody discharge, dimpling, or elevation of one breast. Benign lesions tend to be round or oval with a smooth border and usually show no secondary signs. Furthermore, benign lesions are likely to be movable

Which breast disorder is NOT benign? a) Fibroadenomas b) Fibrocystic breasts c) Duct ectasia d) Paget's disease

d: Paget's disease of the nipple is a rare cancer of the nipple and surrounding skin. Fibrocystic breasts, fibroadenomas, and duct ectasia are all benign conditions of the breast

During a recent visit to the clinic a woman presents with erythema of the nipple and areola on the right breast. She states this started several weeks ago and she was fearful of what would be found. The nurse suspects a) Acute mastitis b) Peau d'orange c) Nipple inversion d) Paget's disease

d: Paget's disease presents with erythema of the nipple and areola. Peau d'orange is associated with breast cancer and is caused by interference with lymphatic drainage. Nipple inversion is considered normal if long-standing; if it is associated with fibrosis and is a recent development, malignancy is suspected. Acute mastitis is associated with lactation but may occur at any age

When counseling a patient on how to prepare for her first mammogram, what should the nurse include? a) "Prepare yourself for a 30- to 45-minute procedure." b) "Schedule the test during or just before your period." c) "Tylenol or Motrin won't help with the discomfort from the procedure." d) "Don't wear any powder, deodorant, or jewelery."

d: Powder, deodorant, and jewelery can distort the images on film and should not be worn. The procedure should be scheduled just after the menses, when the breasts are the least tender. Pain relievers such as acetaminophen and ibuprofen may ease discomfort during the procedure and thereafter. Mammograms are 5- to 10-minute procedures

In which of the follow patients would the nurse first suspect cancer? a) A 30-year-old with a mobile, rubbery, firm, well-circumscribed, nontender lump b) A 25-year-old with multiple small, round, and smooth lesions on both breasts that are painful during menstruation c) A 40-year-old with nipple retraction and a watery discharge d) A 45-year-old with thickening in one breast with nipple irritation and retraction and a pink discharge

d: The 25-year-old most likely has fibrocystic breast changes. The 30-year-old most likely has a fibroadenoma. The 40-year-old most likely has an intraductal papilloma. Further assessment is needed to confirm each of these, but this is what the nurse would first suspect

A female client is diagnosed with breast abscess. Although she has been allowed to breast-feed her newborn, she decides to terminate breast-feeding. Which of the following would be most appropriate in this situation? a) Encourage the client to include protein content in the diet b) Reduce the frequency of removing and reapplying the dressings c) Instruct the client to wear a loose-fitting bra d) Assist the client to pump the breasts to remove breast milk

d: The nurse should help the client pump the breasts and remove breast milk to prevent engorgement. Because the client has decided to terminate breast-feeding, the client should wear a tight-fitting bra. Including protein content in the diet would be unrelated to the client's current situation

A female client who has a 2-month-old infant arrives at a health care facility reporting flu-like symptoms with fever and chills. When examining the breast, the nurse observes an increase in warmth. Which instruction should the nurse provide the client to help her cope with the condition? a) Avoid changing positions while nursing b) Apply cold compresses to the affected breast c) Avoid breastfeeding for 1 month d) Increase fluid intake

d: The nurse should instruct the client with mastitis to increase her fluid intake. A client with mastitis is instructed to continue breastfeeding as tolerated and to frequently change positions while nursing. The nurse should also instruct the client to apply warm, not cold, compresses to the affected breast area or to take a warm shower before breastfeeding

After teaching a group of students about the signs and symptoms of breast cancer, the instructor determines that additional teaching is needed when the group identifies which of the following? a) Nipple retraction b) Painless mass c) Peau d'orange skin d) Breast symmetry

d: The primary sign of breast cancer is a painless mass in the breast. Other signs of breast cancer include a bloody discharge from the nipple, a dimpling of the skin over the lesion, retraction of the nipple, peau d'orange (orange peel) appearance of the skin, and a difference in size between the breasts.

While interviewing a woman before her yearly examination, the nurse learns that she stopped breastfeeding over a year ago but occasionally has pain on one side behind her left nipple not related to her period. Upon examination, the nurse does not palpate any lumps but finds the left nipple retracted with a greenish discharge. What would the nurse first suspect? a) Inflammatory breast cancer b) Paget's disease c) Mastitis d) Duct ectasia

d: These symptoms most closely resemble duct ectasia, which is unilateral, is seen at or near the nipple, and often has a greenish discharge. Cancer cannot be entirely ruled out, but the nurse's first thought should be duct ectasia

A client reports lumpy, tender breasts, particularly during the week before menses. She reports pain that often dissipates after the onset of menses. The nurse suspects the client has fibrocystic breast changes. Which should the nurse do next? a) Have the client follow up in 1 week b) Schedule the client for cryoablation c) Determine if the client has had a mammography d) Perform a breast examination

d: To determine if the client is experiencing fibrocystic breast changes, the nurse must first examine the client's breasts. It is not important to know if the client has a mammography at this time. Cryoabation is done to remove a tumor

A client with breast cancer is to receive immunotherapy. The nurse would expect the client to receive which of the following? a) Raloxifene b) Tamoxifen c) Anastrozole d) Trastuzumab

d: Trastuzumab is the first monoclonal antibody approved for breast cancer treatment. Tamoxifen is a selective estrogen receptor modulator used to prevent further spread of breast cancer in women with ER-positive breast cancer. Raloxifene is a selective estrogen receptor modulator used to prevent further spread of breast cancer; it was originally used solely for prevention and treatment of osteoporosis. Anastrozole is an aromatase inhibitor that interferes with the conversion of androgens to estrogens; it is usually given to women with advanced breast cancer or cancers that recur despite the use of tamoxifen

A nurse is educating a client on the technique for performing breast self-examination. Which of the following instructions should the nurse include in the teaching plan with regard to the different degrees of pressure that need to be applied on the breast?

hard pressure applied near the ribs

The nurse is reviewing the history and physical exam of a woman who has come to the clinic for a routine physical. Which factor would the nurse identify as increasing the client's risk for breast cancer?

history of ovarian cancer A personal history of ovarian cancer is considered a risk factor for breast cancer. Typically, breast cancer is associated with aging (women over 50 years of age). Breast cancer is more common in Caucasian women, but African-American women are more likely to die of it. Early menarche (before 12 years of age) or late onset of menopause (after age 55 years) is associated with an increased risk for breast cancer.

After conducting an in-service program on breast cancer for a group of staff nurses, the leader determines that additional discussion is needed when the group identifies which aspect as influencing the staging of the cancer?

tumor location Breast cancers are classified into three stages based on tumor size, extent of lymph node involvement, and evidence of metastasis. Although location may impact decision making, it is not used for staging purposes.

How should the nurse instruct a woman to perform a breast self-examination? Select all that apply.

• "You will need to visually inspect both breasts and palpate them for changes." • "It doesn't matter which pattern you use to palpate the breast as long as you cover the entire breast." • "Look for any changes in shape, size, contour, or symmetry between the breasts, as well as any skin discoloration."

Lumpectomy is a treatment option for clients diagnosed with breast cancer with tumors smaller than 5 cm. For which of the following clients is lumpectomy contraindicated? Select all that apply.

• Client who has had previous radiation to the affected breast • Client whose connective tissue is reported to be sensitive to radiation • Client whose surgery will not result in a clean margin of tissue

A nurse is assigned to educate a group of women on cancer awareness. Which of the following are the modifiable risk factors for breast cancer? Select all that apply.

• Failing to breastfeed for up to a year after pregnancy • Postmenopausal use of estrogen and progestins • Not having children until after age 30

A woman's breast examination reveals a small mass. Which of the following characteristics would lead the nurse to suspect that the mass is malignant? Select all that apply.

• Hard on palpation • Skin dimpling • Painless

Which factors contribute to the development of breast cancer?

• High breast density • Hormonal factors • Aging • Family history

Which of the following are modifiable risk factors for breast cancer? Select all that apply.

• Smoking • Sedentary lifestyle • Alcohol consumption • Obesity • Duration of breastfeeding

A 38-year-old female client has to undergo lymph node surgery in conjunction with mastectomy. The client is likely to experience lymphedema due to the surgery. Postsurgery, which of the following factors will make the client more susceptible to lymphedema? Select all that apply

• Use of the affected arm for drawing blood or measuring blood pressure • Engaging in activities like gardening without using gloves • Not wearing a well-fitted compression sleeve

Evaluation of a woman with breast cancer reveals that her mass is approximately 1.25 inches in diameter. Three adjacent lymph nodes are positive. The nurse interprets this as indicating that the woman has which stage of breast cancer? A) 0 B) I C) II D) III

C

A woman with breast cancer is undergoing chemotherapy. Which of the following side effects would the nurse interpret as being most serious? A) Vomiting B) Hair loss C) Fatigue D) Myelosuppression

D

When assessing a client with suspected breast cancer, which of the following would the nurse expect to find? A) Painful lump B) Absence of dimpling C) Regularly shaped mass D) Nipple retraction

D

When performing a clinical breast examination, which would the nurse do first? A) Palpate the axillary area. B) Compress the nipple for a discharge. C) Palpate the breasts. D) Inspect the breasts.

D

To help detect potential disease processes in the breasts, the nurse develops educational programs focusing on:

breast awareness. Breast self-exams, once thought essential, are now considered optional. Instead, breast awareness is stressed. This refers to a woman being familiar with the normal consistency of both breasts and the underlying tissue. The emphasis is now on awareness of breast changes, not just discovery of cancer.

A woman diagnosed with breast cancer is to undergo a lumpectomy followed by radiation therapy. When reviewing the treatment plan with the client, the nurse determines that the woman has understood the information based on which client statement?

"After they remove the tumor and some of the normal tissue, I'll start radiation in about 2 to 4 weeks." Women undergoing breast-conserving therapy receive radiation after lumpectomy with the goal of eradicating residual microscopic cancer cells to limit locoregional recurrence. In women who do not require adjuvant chemotherapy, radiation therapy typically begins 2 to 4 weeks after surgery to allow healing of the lumpectomy incision site. Radiation is administered to the entire breast at daily doses over a period of several weeks. Breast-conserving surgery, the least invasive procedure, is the wide local excision (or lumpectomy) of the tumor along with a 1-cm margin of normal tissue.

At a class for new mothers, the nurse notes that the participants need more teaching when one of the mothers makes which comment?

"I have to stop breastfeeding because of mastitis." The nurse should teach the women about the etiology of mastitis and encourage breast-feeding, emphasizing that it is safe for infants. Continued emptying of the breast or pumping improves the outcome, decreases the duration of symptoms, and decreases the incidence of breast abscess.

After teaching a client with mastitis about caring for her breasts, the nurse determines that the client has understood the instructions when she makes which statement?

"I will express milk with a breast pump until infection is resolved." The client statement about expressing milk with a breast pump until the infection is resolved indicates effective teaching. The client should apply warm soaks to the breast or let warm water from a shower flow over the breast. The nurse should advise the client to wear a supportive brassiere. The client should bathe or shower regularly.

After teaching a woman about breast self-examination, the nurse understands that the teaching was successful when the woman makes which statement?

"I'll do the check about a week after my period." Breast self-examination is best performed a week after menses, when swelling has subsided. Breast self-examination is typically performed every month. Both the breast area and the area between the breast and underarm, including the underarm itself, should be part of breast self-examination. The woman should use the pads of her three middle fingers for palpation.

The nurse is giving an educational event for a local women's group on self-examination of the breast. The nurse tells the women's group to raise their arms and inspect their breasts in a mirror. A member of the women's group asks the nurse why she needs to do this. What is the nurse's best response?

"It will help to observe for dimpling." The primary reason for raising the arms is to detect any dimpling. To elicit skin dimpling or retraction that may otherwise go undetected, the examiner instructs the client to raise both arms overhead. This maneuver normally elevates both breasts equally. Pain does not always occur with breast masses. The assessment of the breast should be uniform.

Which pharmacologic treatment would not be therapeutic for a woman with fibrocystic breast changes?

antihistamines Oral contraceptives will keep hormone levels stable to avoid peaks and troughs, which may cause breast edema and pain. Anti-inflammatory medications will be helpful in reducing pain from fibrocystic breast changes. Diuretics can be helpful in reducing excess fluid leading to edema and subsequent pain in the breast. Antihistamines are used to treat an overactive immune response, not edema from breast changes.

It is recommended that a 48-year-old female client with breast cancer undergo a sentinel lymph node biopsy before a lumpectomy. The client asks the nurse the reason for removing the sentinel lymph node. Which statement would the nurse make?

"It will prevent lymphedema, which is a common side effect." The nurse should inform the client that removing only the sentinel lymph node prevents side effects such as lymphedema, which is otherwise associated with a traditional axillary lymph node dissection. It does not help reveal the hormonal status of the cancer. Hormone-receptor status can be revealed through normal breast epithelium, which has hormone receptors and responds specifically to the stimulatory effects of estrogen and progesterone. A sentinel lymph node biopsy will determine how powerful a chemotherapy regimen the client will have to undergo, but undergoing a sentinel lymph node biopsy will not lessen the aggressiveness of the chemotherapy. Degree of HER-2/neu oncoprotein will be revealed through the HER-2/neu genetic marker, not through a sentinel lymph node biopsy.

While performing a clinical breast examination, the nurse notes a firm and rubbery nodule that is well circumscribed and moves freely. How should the nurse counsel the patient?

"It's most likely a fibroadenoma, but we may need to do a biopsy."

While performing a clinical breast examination, the nurse notes a firm and rubbery nodule that is well circumscribed and moves freely. How should the nurse counsel the client?

"It's most likely a fibroadenoma, but we may need to do a biopsy." This description most closely matches a fibroadenoma, but diagnostic imaging and even biopsy are warranted to confirm and rule out a cancerous tumor. The nurse should never tell the client that she may have cancer because this will only cause anxiety.

You are caring for a 52-year-old woman whose sisters and mother died of breast cancer. The patient states, "My doctor wants me to take tamoxifen to help prevent breast cancer. What do you think?" What would be your best response?

"Tamoxifen reduces the incidence."

The nurse is caring for a 52-year-old woman whose sisters and mother died of breast cancer. The client states, "My doctor wants me to take tamoxifen to help prevent breast cancer. What do you think?" What would be the nurse's best response?

"Tamoxifen reduces the incidence." Tamoxifen is referred to as a chemopreventive agent, not a chemotherapeutic agent. It does prevent osteoporosis, but this response does not address breast cancer prevention. Raloxifene is another drug that shows promise as a chemopreventive agent. Tamoxifen reduces breast cancer incidence by 49%.

How should the nurse counsel a postpartum patient on how to prevent mastitis?

"Wash your hands thoroughly, and let your breasts dry after each feeding."

After teaching a group of nursing students about the different types of chemotherapeutic agents used to treat breast cancer, the instructor determines that the teaching was successful when the students identify which of the following as an example of a selective estrogen receptor modulator (SERM)? (Select all that apply.) A) Tamoxifen B) Letrozole C) Raloxifene D) Exemestane E) Anastrozole

A C

A nurse is reviewing the medical records of several female clients. The nurse would identify a woman with which body mass index as being at increased risk for breast cancer?

27 Being overweight or obese is a risk factor for breast cancer in postmenopausal women. Research suggest that women keep a maximum body mass index of 25. Therefore a woman with a BMI of 27 would have an increased risk.

A woman comes to the clinic reporting a nipple discharge. On examination, the area below the areola is red and slightly swollen, with tortuous tubular swelling. The nurse interprets these findings as suggestive of which of the following? A) Fibrocystic breast disorder B) Intraductal papilloma C) Duct ectasia D) Fibroadenoma

C

How often should a woman in her 20s or 30s with no symptoms or family history of breast cancer have a clinical breast examination?

3 The American Cancer Society (ACS) recommends that a woman who is in her 20s or 30s have a clinical breast examination every 3 years.

After finding a breast mass on a routine examination and undergoing diagnostic studies, a woman is diagnosed with a fibroadenoma. The treatment plan focuses on watchful waiting. When would the nurse instruct the client to return for reevaluation?

6 months A client with fibroadenoma being treated with watchful waiting should return for a reevaluation in 6 months, perform monthly self-breast exams, and return annually for a clinical breast exam.

A nurse is preparing a presentation on breast cancer for a local community group. When describing the age group estimated to have the highest risk, which group would the nurse cite?

60 to 69 years of age The age group estimated to be at highest risk for breast cancer is ages 60 to 69 with an estimated risk of 1 out of 29. Those between ages 30 and 39 years have a 1 in 233 chance; those between ages 40 and 49 years have a 1 in 69 chance; and those between ages 50 and 59 have a 1 in 42 chance.

A woman diagnosed with breast cancer is to receive trastuzumab. Which of the following would the nurse incorporate into the explanation about how this drug works? A) It blocks the effect of the HER-2/neu protein inhibiting the growth of cancer cells. B) The drug blocks the conversion of androgens to estrogens C) It interferes with hormone receptors that allow estrogen to enter a cell D) The drug ultimately attacks areas where micrometastasis has occurred.

A

During a clinical breast examination, the nurse palpates a well-defined, firm, mobile lump in a 60-year-old woman's left breast. The nurse notifies the physician. Which of the following would the nurse anticipate the physician to order next? A) Mammogram B) Hormone receptor status C) Fine-needle aspiration D) Genetic testing for BRCA

A

The nurse determines that a woman has implemented prescribed therapy for her fibrocystic breast disease when the client reports that she has eliminated what from her diet? A) Caffeine B) Cigarettes C) Dairy products D) Sweets

A

In which of the follow patients would the nurse first suspect cancer?

A 45-year-old with thickening in one breast with nipple irritation and retraction and a pink discharge

Which of the following is the strongest risk factor for breast cancer? a. Advancing age and being female b. High number of children c. Genetic mutations in BRCA1 and BRCA2 genes d. Family history of colon cancer

A. The incidence of breast cancer increases with aging, especially over age 50. Only 1% of breast cancers occur in men. * C is incorrect: only 7% of women have a genetic mutation resulting in breast cancer, whereas in the remaining 93% it is a sporadic occurrence.

A female client is diagnosed with breast abscess. Although she has been allowed to breast-feed her newborn, she decides to terminate breast-feeding. Which of the following would be most appropriate in this situation?

Assist the client to pump the breasts to remove breast milk

A 33-year-old female client reports yellow nipple discharge and a pain in her breasts a week before menses that dissipates on the onset of menses. Diagnosis reveals that the client is experiencing fibrocystic breast changes. Which instructions should the nurse offer the client to help alleviate the condition? Select all that apply.

Avoid caffeine. Maintain a low-fat diet. Take diuretics as recommended. The nurse should instruct the client with fibrocystic breast changes to avoid caffeine. Caffeine acts as a stimulant that can lead to discomfort. It is important to maintain a lowfat diet rich in fruits, vegetables, and grains to maintain a healthy body weight. Taking diuretics is important to counteract fluid retention and swelling of the breasts. Practicing good handwashing techniques and increasing fluid intake are important for clients with mastitis but may not help clients with fibrocystic breast changes

A client with advanced breast cancer, who has had both chemotherapy and radiation therapy, is to start hormonal therapy. Which agent would the nurse expect the client to receive? A) Progestins B) Tamoxifen C) Cortisone D) Estrogen

B

A laboratory technician arrives to draw blood for a complete blood count (CBC) for a client who had a right-sided mastectomy 8 hours ago. The client has an intravenous line with fluid infusing in her left antecubital space. To obtain the blood specimen, the technician places a tourniquet on the client's right arm. Which action by the nurse would be most appropriate? A) Assist in holding the client's arm still. B) Suggest a finger stick be done on one of the client's left fingers. C) Tell the technician to obtain the blood sample from the client's left arm. D) Call the surgeon to perform a femoral puncture.

B

When describing programs for breast cancer screening, the nurse include breast self-examination (BSE). Which of the following most accurately reflects the current thinking about breast self-examination? A) BSE is essential for early breast cancer detection. B) A woman performing BSE has breast awareness. C) BSE plays a minimal role in detecting breast cancer D) A clinical breast exam has replaced BSE.

B

A woman who has undergone a right modified-radical mastectomy returns from surgery. Which nursing intervention would be most appropriate at this time? A) Ask the client how she feels about having her breast removed. B) Attach a sign above her bed to have BP, IV lines, and lab work in her right arm. C) Encourage her to turn, cough, and deep breathe at frequent intervals. D) Position her right arm below heart level

C

What suggestion would be helpful for the client experiencing painful fibrocystic breast changes? a. Increase her caffeine intake. b. Take a mild analgesic when needed. c. Reduce her intake of leafy vegetables. d. Wear a bra bigger than she needs.

B. The discomfort is usually mild and analgesics will relieve it in most cases.

When preparing for a class on breast cancer, the nurse should explain which nonmodifiable risk factors? Select all that apply.

BRCA-1 gene menarche at age 11 being a 52-year-old female Risk factors can be divided into those that cannot be changed (nonmodifiable) and that that can (modifiable). Nonmodifiable risk factors include gender, age (>50 years old), genetic mutations, personal history of ovarian or colon cancer, early menarche (<12 years old) and race (higher in Caucasian women) just to name a few. Failing to breastfeed, high fat dietary intake, and lack of exercise are examples of modifiable risk factors.

In the United States, what type of cancer accounts for one-third of cancer diagnoses and is the most common cancer in women?

Breast Cancer

A breast biopsy indicates the presence of malignant cells, and the client is scheduled for a mastectomy. Which nursing diagnosis would the nurse most likely include in the client's preoperative plan of care as the priority? A) Risk for deficient fluid volume B) Activity intolerance C) Disturbed body image D) Impaired urinary elimination

C

A woman comes to the clinic and asks the nurse about when she should have her first mammogram. Using the recommendations of the American Cancer Society, which would the nurse suggest? A) 30 years B) 35 years C) 40 years D) 45 years

C

A nurse is working with a woman who has been diagnosed with severe fibrocystic breast disease. When describing the medications that can be used as treatment, which of the following would the nurse be least likely to include? A) Tamoxifen B) Bromocriptine C) Danazol D) Penicillin

D

A postoperative mastectomy client should be referred to which of the following organizations for assistance upon discharge from the hospital? a. National Organization for Women (NOW) b. Food and Drug Administration (FDA) c. March of Dimes Foundation (MDF) d. Reach to Recovery (RTR)

D. This volunteer organization offers support and practical advice to women with breast cancer; all the volunteers have had breast cancer themselves.

The most serious potential adverse reaction from chemotherapy is: a. Thrombocytopenia b. Deep vein thrombosis c. Alopecia d. Myelosuppression

D. When the bone marrow is suppressed secondary to chemotherapy, the woman experiences bleeding tendencies (low platelets), limited immunity (low white blood cells), and anemia (low red blood cells). This myelosuppression can become life-threatening.

During a routine physical examination, a firm mass is palpated in the right breast of a 35-year-old woman. Which finding or client history would suggest cancer of the breast as opposed to fibrocystic disease?

Increased vascularity of the breast

45-year-old woman comes into the OB/GYN clinic for her yearly check-up. The woman mentions to the nurse that she has a dimpling of the right breast that has occurred in the last 2 weeks. She has not performed a self-breast examination. What assessment would be appropriate for the nurse to make?

Palpate the area for a breast mass

During a follow-up visit, a female client who underwent a mastectomy asks the nurse if she can work in her backyard or at least do some household work. Which suggestion would be most appropriate?

Wear gloves and protective clothing to avoid any injuries. The nurse should recommend that the client wear gloves when doing backyard work or housework to prevent injuries that may heal slowly or become infected. Working, whether it be in the backyard or doing some household chores, can be helpful in promoting feelings of usefulness, thereby enhancing the client's coping abilities and self-esteem. She could be advised to follow up more frequently; however, this would not help prevent any untoward injury.

A woman has been treated for a tumor of the left breast that has not responded to chemotherapy. The woman has just found out that she has the BRCA mutations and discusses her options with her primary care provider. Which treatment would be most difficult for this woman?

bilateral mastectomy Chemotherapy, left mastectomy, and radiation therapy may be difficult for the woman, but the most difficult and controversial treatment is bilateral mastectomy. Right mastectomy is considered a prophylactic mastectomy, which is a primary prevention modality. Clients who are considering prophylactic mastectomy are often faced with a very controversial and emotional decision.

A client diagnosed with breast cancer is receiving chemotherapy. The nurse assesses the client for possible side effects of this therapy. Which side effect would the nurse report immediately to the health care provider?

bone marrow suppression Typical side effects include nausea and vomiting, diarrhea or constipation, hair loss, weight loss, stomatitis, fatigue, and immunosuppression. The most serious is bone marrow suppression (myelosuppression). This causes an increased risk of infection, bleeding, and a reduced red blood cell count, which can lead to anemia.

Which factors contribute to the development of breast cancer? Select all that apply.

aging hormonal factors high breast density family history

After teaching a group of students about the signs and symptoms of breast cancer, the instructor determines that additional teaching is needed when the group identifies which sign?

breast symmetry The primary sign of breast cancer is a painless mass in the breast. Other signs of breast cancer include a bloody discharge from the nipple, a dimpling of the skin over the lesion, retraction of the nipple, peau d'orange (orange peel) appearance of the skin, and a difference in size between the breasts.

A nurse is assessing a client who is receiving immunotherapy as part of her treatment plan for breast cancer. The nurse would be alert for which potential side effects? Select all that apply

cardiac toxicity vascular thrombosis Adverse effects of immunotherapy include cardiac toxicity, vascular thrombosis, hepatic failure, fever, chills, nausea, vomiting, and pain with first infusion. Hot flashes, bone pain, and headache are associated with endocrine therapy agents

Which action would the nurse include in the plan of care for a client diagnosed with breast cancer who has undergone a mastectomy?

encouraging the client to perform arm exercises Encouraging the client to perform arm exercises is important to maintain circulation to the arm on the affected side. Use of elastic sleeves, arm pumps, and ace bandaging is recommended for clients who may have complications following radiation or surgery for lymphedema of the arm.

A nurse is assigned to educate a group of women on cancer awareness. Which risk factors for breast cancer are modifiable? Select all that apply.

failing to breastfeed for up to a year after pregnancy postmenopausal use of estrogen and progestins not having children until after age 30 The modifiable risk factors for breast cancer are postmenopausal use of estrogen and progestins, not having children until after the age of 30, and failing to breastfeed for up to a year after pregnancy. Early menarche or late menopause and previous abnormal breast biopsy are the nonmodifiable risk factors for breast cancer.

What is the most common breast mass in women?

fibroadenoma A fibroadenoma is a benign mass in the breast. It is the most common breast mass among women.

A nurse is educating a client on the technique for performing breast self-examination. Which instruction should the nurse include in the teaching plan with regard to the different degrees of pressure that need to be applied on the breast?

hard pressure applied down to the ribs When performing the breast self-examination, the nurse should instruct the client to apply hard pressure down to the ribs. Light, not medium, pressure should be applied when moving the skin without moving the tissue underneath. Medium, not light, pressure should be applied midway into the tissue. The client need not specifically palpate the areolar area during breast self-examination.

During a routine physical examination, a firm mass is palpated in the right breast of a 35-year-old woman. Which finding or client history would suggest cancer of the breast as opposed to fibrocystic disease?

increased vascularity of the breast Increase in breast size or vascularity is consistent with breast cancer. Masses associated with fibrocystic disease of the breast are firm, are most commonly located in the upper outer quadrant of the breast, and increase in size before menstruation. They may be bilateral in a mirror image and are typically well demarcated and freely moveable.

A client is to receive brachytherapy as part of her treatment plan for breast cancer. After teaching the client about this therapy, the nurse determines that additional teaching is needed when the client identifies which condition as a possible side effect?

inflammation Side effects of brachytherapy include redness or discharge around the catheters, fever, and infection. Inflammation is not a side effect.

What is the most common type of breast cancer, accounting for approximately 85% of cases?

invasive ductal carcinoma Invasive ductal carcinoma is the most common type of breast cancer. Invasive lobular carcinoma is the second most common type of breast cancer and accounts for approximately 10% to 15% of all cases.

A client comes to the women's health clinic for a follow-up visit. During the visit, the client says, "I've heard about all these foods that are supposed to have cancer fighting properties. Which foods are these?" The nurse would most likely include which foods in the response? Select all that apply.

leeks cabbage carrots Certain foods demonstrate anticancer properties and boost the immune system. Phytochemical rich foods include: onions and leeks, cooked tomatoes, green leafy vegetables, colorful vegetables such as carrots, and cruciferous vegetables such as cabbage. Almonds are considered a heart-healthy food.

The nurse is providing care to a client who has had surgery as treatment for breast cancer. The nurse would be alert for the development of which complication?

lymphedema Lymphedema occurs in some women after breast cancer surgery. It causes disfigurement and increases the lifetime potential for infection and poor healing. Fibrocystic breast disease and fibroadenoma are two benign breast conditions that occur usually in premenopausal woman. Breast abscess is the infectious and inflammatory breast condition that is common among breastfeeding mothers.

A nurse is preparing a presentation for a health fair about preventing breast cancer. Which suggestion would the nurse include?

maintaining an ideal weight Maintaining an ideal weight decreases the risk of breast cancer. Having no children or having children after age 30 is associated with an increased risk for breast cancer. Some breast tumors are hormone dependent, such that estrogen (or progesterone) enhances tumor growth. Women are advised to avoid the consumption of alcohol, not caffeine, because alcohol correlates with an increased risk of breast cancer.

A client is considering breast augmentation. What would the nurse recommend to the client to ensure that there are no malignancies?

mammogram When caring for a client considering breast augmentation, the nurse should provide her with a general guideline to have a mammogram to verify that there are no malignancies. Mastopexy involves a breast lift for drooping breasts. Ultrasound or breast biopsy would not be necessary unless there was evidence of a problem

A woman is scheduled to undergo a modified radical mastectomy. Which information would the nurse include when describing this surgery to the client?

removal of breast tissue, axillary nodes, and some chest muscles A modified radical mastectomy involves removal of breast tissue, the axillary nodes, and some chest muscles but not the pectoralis major muscle. The surgery will not produce a concave anterior chest. With a simple mastectomy, all breast tissue, the nipple, and the areola are removed, but the axillary nodes and pectoral muscles are spared. A lumpectomy, or breast-conserving surgery, involves the wide local excision of the tumor along with a 1-cm margin of normal tissue.

All options are characteristics of malignant breast disease except:

smooth, firm, rubbery masses. Malignant breast disease is generally characterized by immobile, fixed lumps that may cause skin dimpling and nipple retraction. They are generally painless and not tender. They usually are not smooth and have poorly delineated edges.

As part of a presentation on breast cancer being given to a local woman's group, the nurse describes the need for early detection through screening. Applying the guidelines from the American Cancer Society, the nurse would emphasize which recommendation?

yearly mammograms for women over age 40 The American Cancer Society recommends yearly mammograms for women over age 40. Clinical breast examinations are recommended every year starting at age 40. According to the American Cancer Society Breast Cancer Screening Guidelines, breast self-examination is optional.

A 33-year-old female client complains of yellow nipple discharge and a pain in her breasts a week before menses that dissipates on the onset of menses. Diagnosis reveals that the client is experiencing fibrocystic breast changes. Which of the following instructions should the nurse offer the client to help alleviate the condition? Select all that apply

• Avoid caffeine. • Maintain a low-fat diet. • Take diuretics as recommended


Ensembles d'études connexes

Physical Development in Early Childhood

View Set

Total Rewards (Compensation & Benefits)

View Set

(1) 60s Soul (History of Rock Test 2)

View Set

Insurance - Florida Statutes/Rules/Regulations for Life Insurance

View Set

Amoeba Sisters Video Recap; Photosynthesis and Cellular Respiration

View Set

7.04 Unit Test: Savings Accounts

View Set

Adams Pharmacology Chapter 23-32

View Set

Leadership and Management Questions

View Set